Difference between revisions of "2016 AMC 10B Problems/Problem 18"
(→Solution) |
(→Dumb Solution) |
||
Line 5: | Line 5: | ||
<math>\textbf{(A)}\ 1\qquad\textbf{(B)}\ 3\qquad\textbf{(C)}\ 5\qquad\textbf{(D)}\ 6\qquad\textbf{(E)}\ 7</math> | <math>\textbf{(A)}\ 1\qquad\textbf{(B)}\ 3\qquad\textbf{(C)}\ 5\qquad\textbf{(D)}\ 6\qquad\textbf{(E)}\ 7</math> | ||
− | == | + | ==Solution 1== |
Factor <math>345=3\cdot 5\cdot 23</math>. | Factor <math>345=3\cdot 5\cdot 23</math>. | ||
Line 14: | Line 14: | ||
Looking again at the factors of <math>345</math>, the possible values of <math>k</math> are <math>1,3,5</math> with medians <math>(172,173),(57,58),(34,35)</math> respectively. | Looking again at the factors of <math>345</math>, the possible values of <math>k</math> are <math>1,3,5</math> with medians <math>(172,173),(57,58),(34,35)</math> respectively. | ||
− | Thus the answer is <math>\textbf{( | + | Thus the answer is <math>\textbf{(F) }7</math>. |
==Solution 2== | ==Solution 2== |
Revision as of 20:53, 4 November 2017
Contents
Problem
In how many ways can be written as the sum of an increasing sequence of two or more consecutive positive integers?
Solution 1
Factor .
Suppose we take an odd number of consecutive integers, with the median as . Then with . Looking at the factors of , the possible values of are with medians as respectively.
Suppose instead we take an even number of consecutive integers, with median being the average of and . Then with . Looking again at the factors of , the possible values of are with medians respectively.
Thus the answer is .
Solution 2
We have that we need to find consecutive numbers (an arithmetic sequence that increases by ) that sums to . This calls for the sum of an arithmetic sequence given that the first term is , the last term is and with elements, which is: .
So since it is a sequence of consecutive numbers starting at and ending at . We can now substitute with . Now we subsittute our new value of into to get that the sum is .
This simplifies to . This gives a nice equation. We multiply out the 2 to get that . This leaves us with 2 integers that multiplies to which leads us to think of factors of . We know the factors of are: . So through inspection (checking), we see that only and work. This gives us the answer of ways.
~~jk23541
See Also
2016 AMC 10B (Problems • Answer Key • Resources) | ||
Preceded by Problem 17 |
Followed by Problem 19 | |
1 • 2 • 3 • 4 • 5 • 6 • 7 • 8 • 9 • 10 • 11 • 12 • 13 • 14 • 15 • 16 • 17 • 18 • 19 • 20 • 21 • 22 • 23 • 24 • 25 | ||
All AMC 10 Problems and Solutions |
The problems on this page are copyrighted by the Mathematical Association of America's American Mathematics Competitions.